Last visit was: 26 Apr 2024, 04:05 It is currently 26 Apr 2024, 04:05

Close
GMAT Club Daily Prep
Thank you for using the timer - this advanced tool can estimate your performance and suggest more practice questions. We have subscribed you to Daily Prep Questions via email.

Customized
for You

we will pick new questions that match your level based on your Timer History

Track
Your Progress

every week, we’ll send you an estimated GMAT score based on your performance

Practice
Pays

we will pick new questions that match your level based on your Timer History
Not interested in getting valuable practice questions and articles delivered to your email? No problem, unsubscribe here.
Close
Request Expert Reply
Confirm Cancel
SORT BY:
Date
Math Expert
Joined: 02 Sep 2009
Posts: 92929
Own Kudos [?]: 619124 [27]
Given Kudos: 81609
Send PM
Most Helpful Reply
GMAT Club Legend
GMAT Club Legend
Joined: 12 Sep 2015
Posts: 6818
Own Kudos [?]: 29938 [14]
Given Kudos: 799
Location: Canada
Send PM
General Discussion
Current Student
Joined: 18 Aug 2016
Posts: 531
Own Kudos [?]: 577 [4]
Given Kudos: 198
Concentration: Strategy, Technology
GMAT 1: 630 Q47 V29
GMAT 2: 740 Q51 V38
Send PM
Target Test Prep Representative
Joined: 04 Mar 2011
Status:Head GMAT Instructor
Affiliations: Target Test Prep
Posts: 3043
Own Kudos [?]: 6276 [2]
Given Kudos: 1646
Send PM
Re: Which of the following has the greatest value? [#permalink]
2
Kudos
Expert Reply
Bunuel wrote:
Which of the following has the greatest value?


A. \(\frac{1}{5^37^3}\)

B. \(\frac{6}{5^47^3}\)

C. \(\frac{45}{5^47^4}\)

D. \(\frac{180}{5^57^4}\)

E. \(\frac{900}{5^57^5}\)


We can create a common denominator of (5^5)(7^5) for each answer choice. The choice with the largest numerator will be the fraction with the greatest value:

A)

[1/(5^3)(7^3)][(5^2)(7^2)/(5^2)(7^2)]

Numerator = (5^2)(7^2) = 1225

B)

[6/(5^4)(7^3)][(5^1)(7^2)/(5^1)(7^2)]

Numerator = (6)(5^1)(7^2) = 1470

C)

[45/(5^4)(7^4)][(5^1)(7^1)/(5^1)(7^1)]

Numerator = (45)(5^1)(7^1) = 1575

D)

[180/(5^5)(7^4)][(7^1)/(7^1)]

Numerator = (180)(7^1) = 1260

E)

Numerator = 900

Answer: C
Tutor
Joined: 16 Oct 2010
Posts: 14824
Own Kudos [?]: 64929 [4]
Given Kudos: 426
Location: Pune, India
Send PM
Re: Which of the following has the greatest value? [#permalink]
3
Kudos
1
Bookmarks
Expert Reply
Bunuel wrote:
Which of the following has the greatest value?


A. \(\frac{1}{5^37^3}\)

B. \(\frac{6}{5^47^3}\)

C. \(\frac{45}{5^47^4}\)

D. \(\frac{180}{5^57^4}\)

E. \(\frac{900}{5^57^5}\)


Use the number properties we know:

Whenever we multiply a positive number by another number greater than 1, the number increases. Whenever we multiply it by a number less than 1, the number decreases.
e.g.

2 * (1/2) = 1 (multiplying by 1/2, decreases 2 to 1)

(1/3) * 2 = (2/3) (multiplying by 2 increases 1/3 to 2/3)

A. \(\frac{1}{5^37^3}\)

B. \(\frac{6}{5^47^3}\)

\(= \frac{6}{5} * (A)\) so this is greater than option (A). Eliminate (A)

C. \(\frac{45}{5^47^4}\)

\(= \frac{7.5}{7} * (B)\) so this is greater than option (B). Eliminate (B)

D. \(\frac{180}{5^57^4}\)

\(= \frac{4}{5} * (C)\) so this is less than option (C). Eliminate (D)

E. \(\frac{900}{5^57^5}\)

\(= \frac{20}{35} * (C)\) so this is less than option (C). Eliminate (E)

Answer (C)
Senior Manager
Senior Manager
Joined: 30 Jun 2019
Posts: 275
Own Kudos [?]: 89 [0]
Given Kudos: 8
Send PM
Re: Which of the following has the greatest value? [#permalink]
Take the value presented in A as the base

A) 1
B) increase by 6/5
C) increase by 45/35 = 9/7
D) increase by 180/175
E) decrease by 900/1225

C has the largest increase in change
User avatar
Non-Human User
Joined: 09 Sep 2013
Posts: 32681
Own Kudos [?]: 822 [0]
Given Kudos: 0
Send PM
Re: Which of the following has the greatest value? [#permalink]
Hello from the GMAT Club BumpBot!

Thanks to another GMAT Club member, I have just discovered this valuable topic, yet it had no discussion for over a year. I am now bumping it up - doing my job. I think you may find it valuable (esp those replies with Kudos).

Want to see all other topics I dig out? Follow me (click follow button on profile). You will receive a summary of all topics I bump in your profile area as well as via email.
GMAT Club Bot
Re: Which of the following has the greatest value? [#permalink]
Moderators:
Math Expert
92929 posts
Senior Moderator - Masters Forum
3137 posts

Powered by phpBB © phpBB Group | Emoji artwork provided by EmojiOne